Answer: C
Step-by-step explanation:
X= 1/8(y-1)^2 +4
What are the only two values such that the Fn=n
If 26.97 ÷ 6.3 is written in long division form as so that the divisor is written as a whole number, where should the decimal point be placed in the dividend?
The decimal point should be placed between digits 9 and 7 in the dividend. i.e the dividend becomes 269.7
Step-by-step explanation:Dividend = 26.97 [numerator]
Divisor = 6.3 [denominator]
If 26.97 ÷ 6.3 is written in long division form so that the divisor is written as a whole number, we have the following;
(i)First convert the divisor to a whole number by multiplying by 10 i.e
6.3 x 10 = 63
(ii) Since the divisor (denominator) has been multiplied by 10, to make sure the division expression stays the same, we need to multiply the dividend(numerator) too by 10. i.e
26.97 x 10 = 269.7
(iii) The division expression then becomes;
269.7 ÷ 63
Therefore, the decimal point should be placed between digits 9 and 7 in the dividend.
Evaluate: 5^-2 plsssss
Answer:
0.05
Step-by-step explanation:
5^-2 =
Move the decimal two spaces to the right and you get,
0.05
Hope that helps!
Answer:
the exact form is 1/25 and the decimal form is 0.04 hope dis help
Step-by-step explanation:
What is the to expression?
Answer:
poop
Step-by-step explanation:
trust me its b
P(x)⋅Q(x)=R(x); if P(x)=x+1 and R(x)=2x3+2x2+3x+3, what is Q(x)?
A. 2x2+2
B. 2x2+3
C. 2x2−3
D. 2x2−2
Answer: B
Step-by-step explanation:
\(P(x) Q(x)=R(x) \implies Q(x)=\frac{R(x)}{P(x)}\\\\Q(x)=\frac{2x^{3}+2x^{2}+3x+3}{x+1}={2x^{2}(x+1)+3(x+1)}{x+1}=\boxed{2x^{2}+3}\)
i can’t fail please help
Hey there!
The answer is 6:54
The time on the clock shows 6:20, because the hour hand is after the 6, and the minute hand is on the 4 (which we multiply by 5 to get the minutes).
If it takes her 8 minutes to walk to the bus, 10 to pack lunch, and 16 to get dressed, that's a total of 8 + 10 + 16 minutes, or 34 minutes.
So, we add that to the minutes, so 20 + 34 = 54, therefore the time is 6:54 when she arrives.
Have a terrificly amazing day! :)
Answer:
Yanice should get up at 5:43 to be on time for the bus.
The football players get ready to go to class at 8:17 A.M.
Gymnastics is 50 minutes longer than the piano lessons.
Hope this helps ;)
I need help with this math problem. Thanks!
axy + by + c = 0 (solve for y)
Answer:
\(\displaystyle y = -\frac{c}{ax+b}\)
Step-by-step explanation:
We are given the equation:
\(\displaystyle axy + by + c = 0\)
And we want to solve it for y.
First, isolate all the y-variables. Subtracting c from both sides yields:
\(\displaystyle axy + by = -c\)
We can factor:
\(\displaystyle y(ax + b) = - c\)
And divide. Hence:
\(\displaystyle y = -\frac{c}{ax+b}\)
In conclusion:
\(\displaystyle y = -\frac{c}{ax+b}\)
Answer:
\(axy + by + c = 0 \\ axy + by = - c \\ y(ax + b) = - c \\ \boxed{y = \frac{ - c}{ax + b} }\)
y = -c/(ax+b) is the right answer.1.Suppose a chef ices and decorates cupcakes in batches of 100. Each batch requires 40 minutes to setup the equipment, and each cupcake in the batch takes 1.25 minutes to process. Each unit in the batch must wait for the entire batch to be processed before moving on to packaging. What is the throughput capacity (in cupcakes and/or minutes) of the icing stage? Pick the closest answer.
.6
.8
1
1.25
1.65
2
2. Refer to the previous question. What is the throughput time for a batch of cookies, in minutes? Pick the closest answer.
1.25
2.5
40
125
140
The closest answer is 80 cupcakes per minute, so the correct option is .8. The closest answer is 165 minutes, so the correct option is 165.
The throughput capacity of the icing stage can be calculated by dividing the number of cupcakes in a batch (100) by the time required to process each cupcake (1.25 minutes).
Throughput capacity = Number of cupcakes in a batch / Time to process each cupcake
Throughput capacity = 100 cupcakes / 1.25 minutes
Throughput capacity = 80 cupcakes per minute
The closest answer is 80 cupcakes per minute, so the correct option is .8.
The throughput time for a batch of cupcakes is the time required to process the entire batch, including the setup time.
Throughput time = Time for setup + (Number of cupcakes in a batch * Time to process each cupcake)
Throughput time = 40 minutes + (100 cupcakes * 1.25 minutes per cupcake)
Throughput time = 40 minutes + 125 minutes
Throughput time = 165 minutes
The closest answer is 165 minutes, so the correct option is 165.
Learn more about cupcakes from
https://brainly.com/question/1955459
#SPJ11
Use the following statements to write a compound
statement for the disjunction -p or -q. Then find its truth
value.
p: There are 14 inches in 1 foot.
q: There are 3 feet in 1 yard.
The disjunction of -p or -q can be written as (-p) v (-q). So, we have to find the truth value of (-p) v (-q). So, the compound statement for the disjunction of -p or -q is (-p) v (-q), and its truth value is true.
using the following statements: p: There are 14 inches in 1 foot.
q: There are 3 feet in 1 yard.
Solution: We know that 1 foot = 12 inches, which means that there are 14 inches in 1 foot can be written as 14 < 12. But this statement is false because 14 is not less than 12. Therefore, the negation of this statement is true, which gives us (-p) as true.
Now, we know that 1 yard = 3 feet, which means that there are 3 feet in 1 yard can be written as 3 > 1. This statement is true because 3 is greater than 1. Therefore, the negation of this statement is false, which gives us (-q) as false.
Now, we can use the values of (-p) and (-q) to find the truth value of (-p) v (-q) using the disjunction rule. The truth value of (-p) v (-q) is true if either (-p) or (-q) is true or both (-p) and (-q) are true. Since (-p) is true and (-q) is false, the disjunction of (-p) v (-q) is true. Hence, the compound statement for the disjunction of -p or -q is (-p) v (-q), and its truth value is true.
For more questions on: compound statement
https://brainly.com/question/28794655
#SPJ8
4x + 2 =2x + 10
Please help asap
I will give 15 points
Answer:
6x+12 I think I'm not for sure
Answer:
X=4
Step-by-step explanation:
2X+2=10
2X=8
X=4
please answer this question on algebraic fractions. explain step by step please.
The solution of expression on algebraic fractions is,
⇒ y = 47/21
We have to given that;
Expression to simplify is,
⇒ 5/y + 1/7y - 2/3y = 2
Now, WE can simplify the expression as,
⇒ 5/y + 1/7y - 2/3y = 2
⇒ (35y + y)/7y² - 2/3y = 2
⇒ 36y/7y² - 2/3y = 2
⇒ 36/7y - 2/3y = 2
⇒ (108 - 14) / 21y = 2
⇒ 94 = 21y × 2
⇒ 94 = 42y
Divide both side by 42;
⇒ y = 94 / 42
⇒ y = 47/21
Therefore, The solution of expression on algebraic fractions is,
⇒ y = 47/21
Learn more about the fraction visit:
https://brainly.com/question/5454147
#SPJ1
two cards are selected in a sequence from a standard deck. what is the probability that the second card is a jack given that the first card was a 2. (assume the 2 was not replaced.)
The probability that the second card is a jack given that the first card was a 2 is 52/51.
To calculate the probability that the second card is a jack given that the first card was a 2, we need to consider the remaining cards in the deck after the first card is drawn.
When the first card is drawn and it is a 2, there are 51 cards remaining in the deck, out of which there are 4 jacks.
The probability of drawing a jack as the second card, given that the first card was a 2, can be calculated using conditional probability:
P(Second card is a jack | First card is a 2) = P(Second card is a jack and First card is a 2) / P(First card is a 2)
Since the first card is already known to be a 2, the probability of the second card being a jack and the first card being a 2 is simply the probability of drawing a jack from the remaining 51 cards, which is 4/51.
The probability of the first card being a 2 is simply the probability of drawing a 2 from the initial deck, which is 4/52.
P(Second card is a jack | First card is a 2) = (4/51) / (4/52)
Simplifying the expression:
P(Second card is a jack | First card is a 2) = (4/51) * (52/4)
P(Second card is a jack | First card is a 2) = 52/51
To learn more about probability click here:
brainly.com/question/30736116
#SPJ11
The variables a and b vary directly. Use the given values for a and b to write an equation relating a and b
a=3; b=15
Answer:
a = 1/5 b
Step-by-step explanation:
A direct variation is given by
a = k*b where k is the constant of variation
3 = k* 15
Divide each side by 15
3/15 = k*15/15
1/5 = k
a = 1/5 b
Answer:
a = 1/5 b
Step-by-step explanation:
Which of the following answer choices shows which numbers from the list below are less than - 0.94?
Answer:
-2.23, -2 2/5, -0.95
Step-by-step explanation:
-15/16 = -0.9375 (greater than -0.94)
0.24 (greater than -0.94)
-2.23 (less than -0.94)
97% = 0.97 (greater than -0.94)
-2 2/5 = -2.4 (less than -0.94)
-0.95 (less than -0.94)
can you help me please
Answer:
r = 728 ( there might be an easier way! )
Step-by-step explanation:
w/r = 3/8 then w = 3/8 r
b/w = 2/7 b = 2/7 w sub in the above equation b = 2/7 (3/8 r) = 6/56 r
r + w + b = 1079
r + 3/8 r + 6/56 r = 1079
r = 728
WILL GIVE BRAINLIEST!
To override a presidential veto, at least 66 2/3% of the Senate must vote to override the veto. There are 100 senators in the Senate. What is the minimum number of votes needed to override a veto? (Round up to the nearest vote.)
Answer:
I'm sure the answer is 34 8/10
Evaluate
What is the product of 12 and -5 separated into 6 equal parts?
Ans -10.
Step-by-step explanation:.
product means multiply and separate means divide, 12 x -5 = -60/6 = -10
1.14 Suppose 8-bit digital encoding is used to represent the angle between 0^{\circ} and 180^{\circ} where 00{~h} represents 0^{\circ} and {FFh} represents
Given that 8-bit digital encoding is used to represent the angle between 0° and 180° where 00h represents 0° and FFh represents 180°.
Hence, each bit of the digital encoding represents a specific fraction of the range between 0° and 180°.The fraction represented by each bit is 1/128 since 2^7 = 128.
Therefore, the resolution of the encoding is 1/128 of 180°, which is approximately 1.406°
.The angle represented by any particular 8-bit code word is determined by multiplying the code word by the fraction that each bit represents and adding up the results.
Since each bit represents 1/128 of 180°, the angle represented by a code word is given by the formula θ = (code word)/128 × 180°.For example, the code word 00101010 represents (00101010)16 = (42)10, and hence the angle represented by this code word is
\(θ = (42/128) × 180° = 59.06°.\)
Therefore, 8-bit digital encoding can be used to represent angles with a resolution of approximately 1.406° and an accuracy of 1.406°.
To know more about digital visit:
https://brainly.com/question/15486304
#SPJ11
There are 16 boys, and 14 girls in Mr. Smith's homeroom. If two students are absent, what is
the probability that both absent students are boys?
Answer:
2/16
simplest form:1/8
Step-by-step explanation:
2/16
simplest form:1/8
There are 16 boys in all so 16 will be your denominator and there are to boys absent so your numerator
2/16
Then simplify by dividing 2/2
1/8
(c) Verify that y = cos x is a solution to this differential equation. =
1. Consider the Differential Equation: -yy" + (y')2 = 1 = (a) The order of this equation is: (b) Decide whether this equation
The given differential equation is -yy" + (y')^2 = 1. The order of this equation is second order.
To verify whether y = cos(x) is a solution to this differential equation, we need to substitute y = cos(x) into the equation and check if it satisfies the equation.
The order of a differential equation is determined by the highest derivative present in the equation. In this case, the highest derivative is y", so the order of the equation is second order.
To verify if y = cos(x) is a solution to the differential equation, we substitute y = cos(x) into the equation:
-(cos(x))(cos''(x)) + (cos'(x))^2 = 1.
Taking the derivatives, we have:
cos'(x) = -sin(x) and cos''(x) = -cos(x).
Substituting these values into the equation, we get:
-(cos(x))(-cos(x)) + (-sin(x))^2 = 1.
Simplifying the equation, we have:
cos^2(x) + sin^2(x) = 1.
Since cos^2(x) + sin^2(x) = 1 is an identity, it is true for all values of x. Therefore, y = cos(x) is indeed a solution to the given differential equation.
Learn more about derivative here:
https://brainly.com/question/29020856
#SPJ11
Consider the function f(x, y) = 4 sin (5x - 6y) and the point P(0, 2 pi). a. Find the unit vectors that give the direction of steepest ascent and steepest descent at P.
These unit vectors give the direction of steepest ascent and steepest descent at point P(0, 2π) for the function f(x, y) = 4sin(5x - 6y).
To find the unit vectors that give the direction of steepest ascent and steepest descent at point P(0, 2π) for the function f(x, y) = 4sin(5x - 6y), we need to calculate the gradient vector at that point.
The gradient vector represents the direction of the steepest ascent, and its negative represents the direction of the steepest descent. The gradient vector is given by the partial derivatives of the function with respect to x and y, multiplied by a scalar factor:
∇f(x, y) = (4(5)cos(5x - 6y), -4(6)cos(5x - 6y))
Evaluating the gradient vector at point P(0, 2π), we get:
∇f(0, 2π) = (4(5)cos(0 - 12π), -4(6)cos(0 - 12π))
= (20cos(-12π), -24cos(-12π))
To obtain the unit vectors in the direction of steepest ascent and steepest descent, we divide the gradient vector by its magnitude:
Unit vector of steepest ascent = ∇f(0, 2π) / ||∇f(0, 2π)||
Unit vector of steepest descent = -∇f(0, 2π) / ||∇f(0, 2π)||
Calculating the magnitudes:
||∇f(0, 2π)|| = sqrt((20cos(-12π))^2 + (-24cos(-12π))^2)
Finally, we divide the gradient vector by its magnitude to obtain the unit vectors:
Unit vector of steepest ascent = (∇f(0, 2π) / ||∇f(0, 2π)||) = (20cos(-12π) / ||∇f(0, 2π)||, -24cos(-12π) / ||∇f(0, 2π)||)
Unit vector of steepest descent = (-∇f(0, 2π) / ||∇f(0, 2π)||) = (-20cos(-12π) / ||∇f(0, 2π)||, 24cos(-12π) / ||∇f(0, 2π)||)
These unit vectors give the direction of steepest ascent and steepest descent at point P(0, 2π) for the function f(x, y) = 4sin(5x - 6y).
Know more about Steepest here :
https://brainly.com/question/11926421
#SPJ11
How many powers of 7 are contained in 10?
There is only 1 power of 7 contained in 10.
Students use powers of numbers to solve this problem and learn what is occurring to the numbers as a result.
Students also learn how to divide a seemingly vast and challenging equation into smaller, more accessible components. The pupils should understand that raising 7 to a power only produces a finite number of unit digits. Furthermore, as the power of 7 rises, these particular numbers "circle round." 7, 9, 3, and 1 make up this cycle.
The digit in the tens place is the same.
The total number of times we multiply a number is known as its exponent or power. For instance, 2 to the power 3 denotes a 3x3 multiplication of 2.
The largest power of 7 that is less than 10 is 7^1 = 7, therefore there is only 1 power of 7 contained in 10.
To know more about exponents visit :
https://brainly.com/question/30066987?referrer=searchResults
#SPJ4
There is only 1 power of 7 contained in 10.
Students use powers of numbers to solve this problem and learn what is occurring to the numbers as a result.
Students also learn how to divide a seemingly vast and challenging equation into smaller, more accessible components. The pupils should understand that raising 7 to a power only produces a finite number of unit digits. Furthermore, as the power of 7 rises, these particular numbers "circle round." 7, 9, 3, and 1 make up this cycle.
The digit in the tens place is the same.
The total number of times we multiply a number is known as its exponent or power. For instance, 2 to the power 3 denotes a 3x3 multiplication of 2.
The largest power of 7 that is less than 10 is 7^1 = 7, therefore there is only 1 power of 7 contained in 10.
To know more about exponents visit :
brainly.com/question/30066987?referrer=searchResults
#SPJ4
5. Find the Fourier coefficients of the periodic ( -5 to 5) function y(t) = -3 when -5
In summary, the Fourier coefficients for the periodic function y(t) = -3 on the interval -5 ≤ t ≤ 5 are:
c₀ = -3 (DC component)
cₙ = 0 for n ≠ 0 (other coefficients)
To find the Fourier coefficients of the periodic function y(t) = -3 on the interval -5 ≤ t ≤ 5, we can use the formula for Fourier series coefficients:
cn = (1/T) ∫[t₀-T/2, t₀+T/2] y(t) \(e^{(-i2\pi nt/T)}\) dt
where T is the period of the function and n is an integer.
In this case, the function y(t) is constant, y(t) = -3, and the period is T = 10 (since the interval -5 ≤ t ≤ 5 spans 10 units).
To find the Fourier coefficient c₀ (corresponding to the DC component or the average value of the function), we use the formula:
c₀ = (1/T) ∫[-T/2, T/2] y(t) dt
Substituting the given values:
c₀ = (1/10) ∫[-5, 5] (-3) dt
= (-3/10) \([t]_{-5}^{5}\)
= (-3/10) [5 - (-5)]
= (-3/10) [10]
= -3
Therefore, the DC component (c₀) of the Fourier series of y(t) is -3.
For the other coefficients (cₙ where n ≠ 0), we can calculate them using the formula:
cₙ = (1/T) ∫[-T/2, T/2] y(t)\(e^{(-i2\pi nt/T) }\)dt
Since y(t) is constant, the integral becomes:
cₙ = (1/T) ∫[-T/2, T/2] (-3) \(e^{(-i2\pi nt/T)}\) dt
= (-3/T) ∫[-T/2, T/2] \(e^{(-i2\pi nt/T)}\) dt
The integral of e^(-i2πnt/T) over the interval [-T/2, T/2] evaluates to 0 when n ≠ 0. This is because the exponential function oscillates and integrates to zero over a symmetric interval.
all the coefficients cₙ for n ≠ 0 are zero.
To know more about function visit:
brainly.com/question/30721594
#SPJ11
a rotcaf is the number that results from adding a number and the greatest factor of the number that is less than the number. for example, the rotcaf of 9 is 9 3, or 12. what is the sum of the rotcafs of all the even numbers between 1 and 25?
234 is the sum of the rotcaf of all the even numbers between 1 and 25.To solve this question use the concept of series.
What is rotcaf?A rotcaf is the sum of the largest factors of the number that is smaller than the number and the number that is obtained by adding two numbers, the rotcaf of 9, for instance, is 9, 3, or 12.
Here given that,
even numbers between 1 and 25 which is: 2,4,6,8,10,12,14,16,18,20,22,24
for finding rotcaf:
(2 + 1) , (4 + 2), (6 + 3), (8 + 4),.....(24 + 12)
Here two series are present:
(1, 3, 5, 7,......23) and (2,4,6,8,......24)
Now, sum of the series is:
= [12 /2 {2+24}] + [12/2 {1 + 12}
= 156 + 78
= 234
To know more about series refer to:
https://brainly.com/question/6561461
#SPJ4
Find (if possible) the rational zeros of the function. (Enter your answers as a comma-separated list. If an answer does not exist, enter DNE.)
C(x) = 2x3 + 3x2 − 1
Therefore, the possible rational zeros of C(x) are -1/2, 1/2, -1, 1, -2, and 2.
To find the rational zeros of the function C(x), we need to find the factors of the constant term -1 and the factors of the leading coefficient 2. Then we can form all possible fractions that can be obtained by dividing the factors of the constant term by the factors of the leading coefficient.
The factors of -1 are ±1 and ±1/2, and the factors of 2 are ±1 and ±2. Therefore, the possible rational zeros of C(x) are:
±1/2, ±1, ±1/2, ±1/1, ±2/2
Simplifying these fractions, we get:
±1/2, ±1, ±2
To know more about possible rational zeros,
https://brainly.com/question/31054413
#SPJ11
The method of separable ODEs can be applied only when the right-hand side of an ODE dy/dt=f(y,t) can be rewritten as the sum of a function of y alone and a function of t alone.
a. true b. false
The right-hand side of an ODE dy/dt=f(y,t) can be rewritten as the product of a function of y alone and a function of t alone.
False.
The method of separable ODEs can be applied when the right-hand side of an ODE dy/dt = f(y,t) can be written as a product of a function of y alone and a function of t alone, not necessarily as the sum of such functions. Specifically, the ODE can be written in the form:
g(y) dy/dt = h(t)
where g(y) is a function of y only and h(t) is a function of t only.
We can then integrate both sides with respect to their respective variables to obtain:
∫ g(y) dy = ∫ h(t) dt + C
where C is the constant of integration. We can then solve for y in terms of t, if possible, to obtain the general solution of the ODE.
Therefore, the correct statement is: The method of separable ODEs can be applied only when the right-hand side of an ODE dy/dt=f(y,t) can be rewritten as the product of a function of y alone and a function of t alone.
To learn more about statement visit:
https://brainly.com/question/10705953
#SPJ11
The scale drawing is a map of the campus of Central Middle School. The school board needs to know
the distance from the art room to the gym in order to build a walkway.
What is the scale factor from the map to the
actual school?
scale factor =?
Answer:Scale factor is 10 , the second part is 50
Step-by-step explanation:
three points t, u, and v on the number line have coordinates t, u, and v, respectively. is point t between points u and v ?
We can determine coordinates if point t is between points u and v by checking if u < t < v or v < t < u.
To determine if point t is between points u and v, we need to compare their coordinates. If u < v, then point t is between points u and v if and only if u < t < v. On the other hand, if v < u, then point t is between points u and v if and only if v < t < u.
Whether or not point t is between points u and v depends on the relationship between the coordinates of u and v. If u < v, t must fall between them, and if v < u, t must also fall between them.
To know more about coordinates follow the link:
https://brainly.com/question/31306451
#SPJ11
What is the length of the line?
Answer:
6 square units
Una jarra contiene medio litro de gaseo-
sa, ¿cuántos vasos de 10 centilitros se
pueden llenar?
5 vasos ya que en u litro cabe 100 entonces en medio ay 50 y 10 cabe 5 veses en el 50